4th moment, show that E[X-mu]^4 is equal to

  • Thread starter Thread starter alias
  • Start date Start date
  • Tags Tags
    Moment
Click For Summary
The discussion revolves around proving the fourth moment of a random variable X, specifically showing that E[X-mu]^4 equals a derived expression involving E(X), E(X^2), E(X^3), and E(X^4). Participants express confusion about the necessity of proving the result separately for discrete and continuous cases, with one noting that the same equation applies to both. The challenge lies in expanding (X-mu)^4 using the binomial theorem and understanding the specific summation and integral required for each case. Ultimately, the consensus is that while a split is not necessary, the question demands separate proofs. Clarification on the approach to take for each case remains a point of concern.
alias
Messages
40
Reaction score
0

Homework Statement


X is a random variable with moments, E[X], E[X^2], E[X^3], and so forth. Prove this is true for i) X is discrete, ii) X is continuous


Homework Equations


E[X-mu]^4 = E(X^4) - 4[E(X)][E(X^3)] + 6[E(X)]^2[E(X^2)] - 3[E(X)]^4
where mu=E(X)

The Attempt at a Solution


I've been trying to generalize expanding the variance, E[(X-mu)^2], into the above result with no success. Not sure about the discrete and continuous proofs either. Anyone have any ideas?
 
Physics news on Phys.org
EX = m is a number, not a random quantity. So, expand (X-m)^4 using the binomial theorem.

RGV
 
I got the expansion, thanks. I'm not sure why I need to split the cases for continuous/discrete either. It's asked in the question and I don't know how to prove with continuous/discrete separately.
 
alias said:
I got the expansion, thanks. I'm not sure why I need to split the cases for continuous/discrete either. It's asked in the question and I don't know how to prove with continuous/discrete separately.

No such split is needed.

RGV
 
Ray Vickson said:
No such split is needed.

RGV

I know that the one equation will cover both the discrete and continuous cases, but the question specifically asks to show each case individually and I'm not sure what the specific summation and integral that I have to work out is.
 
Question: A clock's minute hand has length 4 and its hour hand has length 3. What is the distance between the tips at the moment when it is increasing most rapidly?(Putnam Exam Question) Answer: Making assumption that both the hands moves at constant angular velocities, the answer is ## \sqrt{7} .## But don't you think this assumption is somewhat doubtful and wrong?

Similar threads

  • · Replies 10 ·
Replies
10
Views
2K
  • · Replies 5 ·
Replies
5
Views
2K
  • · Replies 4 ·
Replies
4
Views
2K
Replies
2
Views
2K
  • · Replies 6 ·
Replies
6
Views
2K
Replies
7
Views
1K
  • · Replies 4 ·
Replies
4
Views
2K
  • · Replies 13 ·
Replies
13
Views
2K
  • · Replies 7 ·
Replies
7
Views
2K
  • · Replies 7 ·
Replies
7
Views
2K